6 votos

Máximo y mínimo de $\frac{1}{n} \cot(n \pi \phi)$ , $\phi$ Relación áurea

Estudiar aspectos de este problema He tropezado con esta pregunta.

Designación de la ración de oro por $\phi=\frac{1+\sqrt{5}}{2} \simeq 1.61803$ y dejar que

$$a(n) = \frac{1}{n} \cot(n \pi \phi)$$

(i) demostrar que $a(n)$ está limitada por arriba y por abajo

(ii) calcular $\max (a(n))$ y $\min (a(n))$ con $n =1,2,3,...$

(iii) resolver el problema similar cuando $\cot$ se sustituye por $\csc$ , es decir, considerar

$$b(n) = \frac{1}{n \sin(\pi \phi n)}$$

En este caso comprueba la validez de mi conjetura de que $b(1)< b(n) < b(3)$ para $n\gt 3$

(iv) Extensión: lo mismo si $\phi$ se sustituye por otras cantidades irracionales como $\sqrt{2}$ , $2^{\frac{1}{3}}$ , $\log{2}$ , $\gamma$ , $\pi$ , $e$ . Aquí, excepto en el caso $\sqrt{2}$ No tengo ningún indicio de que los extremos existan, es decir, que $a(n)$ está acotado si $n \to \infty$ .

¿Qué hace que esta pregunta sea interesante (IMHO)?

Un aspecto es este:

La expresión $b(n)$ cuando se considera en función de real $n\gt 0$ tiene polos simples en

$$n_{k} = k/ \phi, k=1,2,3,...$$

Un número entero $n$ puede llegar a estar muy cerca de un $n_k$ . Me pareció sorprendente que el factor de amortiguación más bien modesto $\frac{1}{n}$ es capaz de anular la fuerte subida en las proximidades de los polos.

¿Qué he hecho hasta ahora?

La parte modesta que hice hasta ahora está en la referencia anterior. Además, aquí están los gráficos de las cantidades en cuestión. Observación: la elección de los números de Fibonacci como límite superior del rango se hace plausible en la investigación citada.

0 votos

Puede ser más fácil buscar alguna información sobre los límites de los valores absolutos de esas secuencias. En ese caso, las medidas de irracionalidad están relacionadas.

0 votos

Así que para los irracionales cuadráticos o los números con cocientes parciales acotados esas secuencias están acotadas.

0 votos

@ i707107 Esto suena interesante. ¿Podría explicar algo más, preferiblemente en una respuesta?

1voto

Para un número irracional con cociente parcial acotado, tenemos lo siguiente.

Si $\theta$ es un número irracional con cocientes parciales acotados, es decir, la expansión de fracción continua de $\theta=[a_0;a_1,a_2,\ldots]$ tiene $a_n\leq K, \ n\geq 1$ para algunos fijos $K>0$ . Entonces, para cualquier número entero $p, q$ con $q>0$ tenemos una constante absoluta $c=c(\theta)>0$ tal que $$ \left| \theta-\frac pq \right| \geq \frac 1{cq^2}. $$

Entonces tenemos $k\|k\theta\|\geq c$ para cualquier número entero positivo $k$ donde $\|x\|$ es la distancia al número entero más cercano a $x$ . Desde $| \cot k\theta | = \frac1{\pi\|k\theta\|} + O(1)$ y $|\csc k\theta|=\frac1{\pi \|k\theta\|}+O(1)$ tenemos

$|\cot k\theta|/k\leq 1/(c\pi)+O(1)$ y $|\csc k\theta|/k\leq 1/(c\pi)+ O(1)$ para cualquier número entero $k>0$ .

Esto demuestra la acotación de las dos secuencias en el caso $\theta = \phi$ , $\sqrt 2$ ya que son irracionales cuadráticos, que tienen cocientes parciales acotados.

Para otros números, no hay información suficiente para concluir la acotación. Ver también este post en MathOverflow: https://mathoverflow.net/questions/224340/is-there-any-pattern-to-the-continued-fraction-of-sqrt32

Véase también este post mío que utilizó la misma técnica. Hace $\sum_{k=1}^n|\cot \sqrt2\pi k|$ tiende a $An\ln n$ como $n\to\infty$ ?

0 votos

@ i707107 Tu solución y tu post citado llenaron el vacío que encontré al tratar de usar el teorema de Roth en la forma $\left| \theta-\frac pq \right| \geq \frac 1{cq^{2+\epsilon}}$ . Porque no pude ver que $\epsilon = 0$ para los números algéricos en cuestión. También he aprendido que pertenecen a la clase de "números irracionales con cocientes parciales acotados" (un concepto que era nuevo para mí)..

i-Ciencias.com

I-Ciencias es una comunidad de estudiantes y amantes de la ciencia en la que puedes resolver tus problemas y dudas.
Puedes consultar las preguntas de otros usuarios, hacer tus propias preguntas o resolver las de los demás.

Powered by:

X